An Infinite Tower

Поділитися
Вставка
  • Опубліковано 5 січ 2024
  • 🤩 Hello everyone, I'm very excited to bring you a new channel (aplusbi)
    Enjoy...and thank you for your support!!! 🧡🥰🎉🥳🧡
    / @sybermathshorts
    / @aplusbi
    ⭐ Join this channel to get access to perks:→ bit.ly/3cBgfR1
    My merch → teespring.com/stores/sybermat...
    Follow me → / sybermath
    Subscribe → ua-cam.com/users/SyberMath?sub...
    ⭐ Suggest → forms.gle/A5bGhTyZqYw937W58
    If you need to post a picture of your solution or idea:
    intent/tweet?text...
    #radicals #infinity #sequencesandseries
    via @UA-cam @Apple @Desmos @NotabilityApp @googledocs @canva
    PLAYLISTS 🎵 :
    Number Theory Problems: • Number Theory Problems
    Challenging Math Problems: • Challenging Math Problems
    Trigonometry Problems: • Trigonometry Problems
    Diophantine Equations and Systems: • Diophantine Equations ...
    Calculus: • Calculus

КОМЕНТАРІ • 27

  • @williamperez-hernandez3968
    @williamperez-hernandez3968 5 місяців тому +21

    The chain rule was incorrectly calculated. Should be: 1/x^2 - (ln x)/x^2 = (1 - ln x )/x^2. So the result of extreme ocurring at x=e does not change.

  • @dariosilva85
    @dariosilva85 5 місяців тому +11

    The function y=x^(1/x) can be calculated for all x between 0+ and infinity. You stated that the original equation has only solutions when x is between (1/e) and e. How did you arrive at that?

  • @maxgoldman8903
    @maxgoldman8903 5 місяців тому +5

    For the sake of clarity, you should state the convergence conclusion is derived from the Banach fixed point theorem.

  • @debasishsarkar5000
    @debasishsarkar5000 5 місяців тому

    I regularly see your videos and learn many complex way of solution.

  • @angelishify
    @angelishify 5 місяців тому

    The sequence x_n+1= y^x_n has 2 stationary points for 1 < y < e^ (1/e) = 1.445 at x_l < e and x_u > e.
    Derivative of the function (y^x)' = ln y ⋅ y^x = ln x (for y^x = x at the stationary point).
    This is less than one for x_l and greater then one for x_u.The lower x_l is therefore a stabile stationary point and the upper x_u is a labile stationary point.
    If we start our sequence at x_1 = y (which is our issue, y = 3^(1/3) = 1.442) it will end up in the attractive lower stationary point x_l = 2.478.

  • @mrityunjaykumar4202
    @mrityunjaykumar4202 5 місяців тому +1

    if we see intuitively the cube root of 3 at the very top exponent ( at infinity) then (cube root 3)^(cube root 3)>(cube root 3)...now coming down the exponent lane, each of the exponent will be larger than the previous one, hence finally we will have (cube root 3) raise to power infinite which will be infinite.. it's definitely a divergent series..

  • @MathZoneKH
    @MathZoneKH 5 місяців тому

    You always have the best way and cold problem❤

  • @teambellavsteamalice
    @teambellavsteamalice 5 місяців тому

    Why then does the answer x = 3 work?
    One step y^3 gives (3^(1/3))^3 = 3^(1/3*3) = 3, so you can repeat that infinitely.
    Maybe y needs to be between 1/e and e for it to converge, but x doesn't?
    If so then all y = a^(1/a) could work, and for any problem y^y^y^... = x the answer would be x = a.

  • @MisterPenguin42
    @MisterPenguin42 5 місяців тому +1

    What introduced the extraneous solution?

    • @Debg91
      @Debg91 5 місяців тому +1

      When you assume convergence, you may do some calculation that isn't justified. Just because you end up with a finite result, doesn't automatically imply that the quantity converged in the first place; this is why you should always check for convergence first before calculating an infinite sequence.

    • @MisterPenguin42
      @MisterPenguin42 5 місяців тому +1

      @@Debg91 appreciate you!!

  • @Qermaq
    @Qermaq 5 місяців тому +1

    Fun one. What's the minimum and maximum a among the reals where a^a^a^a^... = x and x is a defined value?

    • @user-asdlfhadfhal
      @user-asdlfhadfhal 5 місяців тому

      I remember video of sybermath about it. a^a^a^…=x, so a^x=x. so a=x^1/x.

    • @Qermaq
      @Qermaq 5 місяців тому

      @@user-asdlfhadfhal Yes, but we know 2^2^2^2^... is going to diverge. 2^x = x. Well 2^2 = 4, 2^1 = 2, 2^0 = 1, 2^0.5 = 1.414..., see they can never be equal. The exponent is always less than the result. So a = 2 diverges.
      So what's the largest real a that won't diverge?

    • @user-asdlfhadfhal
      @user-asdlfhadfhal 5 місяців тому

      @@Qermaq hmm… I think you are smart than me

    • @user-asdlfhadfhal
      @user-asdlfhadfhal 5 місяців тому +1

      @@Qermaq there is no root about 2^x=x, so a’s value that make the equation a^x=x has root is a that y=a^x and x are contact in line, so let’s say the point that both encounter is (t,a^t). differential coefficient of a^x at t is ln a*a^t, it’s same with differential coefficient of x, which is 1. So a^t=t, ln a*a^t=1. by calculation, t*ln a=1, so a^t=e. and a^t was t, so t=e.
      So, a^e=e, so a=e^1/e, if a is larger than e^1/e, both won’t be in contact, and if less, both will be in contact at two point. So largest a is e^1/e??

    • @user-asdlfhadfhal
      @user-asdlfhadfhal 5 місяців тому

      @@Qermaq To ask for an excuse, I’m bad at English

  • @angelamusiemangela
    @angelamusiemangela 5 місяців тому

    0,3?

  • @user-lu6yg3vk9z
    @user-lu6yg3vk9z 5 місяців тому

    Try this problem
    Integral 1/(1+(sin(x))^2) dx
    Hint multiple top and bottom sec(x)^2

  • @user-fq4gu3gm9z
    @user-fq4gu3gm9z 5 місяців тому

    3

  • @TedHopp
    @TedHopp 5 місяців тому

    Towards the end of the video, you recast the problem as finding the limit of a sequence: y, y^y, y^(y^y), .... You really need to show that this sequence converges before any of the previous analysis can be accepted. This was never done and there's no reason to think that the "correct" result isn't as spurious as 3. Just because it falls within the extreme values for the equation y^x=x isn't enough.

  • @giuseppemalaguti435
    @giuseppemalaguti435 5 місяців тому

    x=e^(-W(-a))=2,4777...a=(1/3)ln3

    • @altosrule6387
      @altosrule6387 5 місяців тому

      I derived x=W(-a)/(-a), equivalent to your result because W(-a)/(-a)=e^(-W(-a)) is implied by W(-a)*e^(W(-a))=(-a), which is implied by W(W(-a)*e^(W(-a)))=W(-a).